LSAT and Law School Admissions Forum

Get expert LSAT preparation and law school admissions advice from PowerScore Test Preparation.

 Administrator
PowerScore Staff
  • PowerScore Staff
  • Posts: 8916
  • Joined: Feb 02, 2011
|
#24700
Complete Question Explanation

Weaken. The correct answer choice is (C)

The author of this stimulus is skeptical regarding the prospect of computers replacing teachers. If teaching were simply a transfer of facts and rules, then the computer could do a decent job. But, the author asserts, understanding is more than just facts and rules, but also the grasp of underlying concepts. Therefore, the author concludes, the computer will never replace the teacher. The argument’s basic components are as follows:
  • Premise: ..... Computers can relay basic facts and rules.

    Premise: ..... But understanding requires a grasp of underlying concepts.

    Conclusion: ..... Therefore computers will never be able to teach full understanding.
The author clearly believes that computers are unable to teach a grasp of underlying concepts (if the question stem were a justify question or a supporter assumption question, that would be the answer). This stimulus is followed by a weaken question, so the correct answer choice might rule out the author’s implied belief about computers’ limitations.

Correct choice (C) provides the prephrased weakening answer: if it is possible to program computers to teach an understanding of underlying concepts, this significantly weakens the author’s argument.
 andriana.caban
  • Posts: 142
  • Joined: Jun 23, 2017
|
#71702
Hi!

So I had trouble deciding between (A) and (E). Ultimately I eliminated (E) because the author of the stimulus discusses grasping "general concepts" and answer choice broadens this by saying "concepts" I figured it was a shell game answer. Also, if (E) were true, wouldn't it completely destroy the authors argument. If students could not develop an understanding of the concepts (disregarding the "general concepts" v "concepts" issue) underlying facts and rules through practice and drills, what would be the point in having teachers. In fact, there would be no point in having computers at all.

(E) gives us a scenario where neither teachers nor computers can be adequate to teach / instruct students. In a way it strengthens the authors claim that computers won't replace teachers but also weakens to a degree that goes beyond the task at hand (because we want to show that computers COULD replace teachers).

Please let me know if my reasoning is correct or if I'm going too far.

Thank you so much guys!
 Adam Tyson
PowerScore Staff
  • PowerScore Staff
  • Posts: 5153
  • Joined: Apr 14, 2011
|
#72367
I think you're missing the underlying assumptions in this argument, andriana. The author must believe that teachers CAN, in addition to conveying rules and facts, also convey the underlying concepts (and there is no need to distinguish here between "general concepts" and "concepts"). And the author must also assume that computers CANNOT convey those concepts. Thus, to weaken this argument, we either want an answer that says teachers CANNOT do that or that computers CAN do that. If either of those statements were true, then it might be possible for a computer to replace a teacher.

That's why C is the correct answer to this question. If a computer can be programmed to convey the underlying concepts, then one of the author's key assumptions is false, and the argument falls apart.

Answer A does nothing to hurt the argument because it does not address the degree to which a computer might be able to convey those concepts. Answer E might actually strengthen the argument, because it means that a computer that only does drills and coaching will fail to teach the required concepts.

Get the most out of your LSAT Prep Plus subscription.

Analyze and track your performance with our Testing and Analytics Package.